The Assignment Method: Definition, Applications, and Implementation Strategies

Last updated 03/15/2024 by

Fact checked by

Understanding the assignment method

Optimized resource utilization, enhanced production efficiency, maximized profitability, applications of the assignment method, workforce allocation, production planning, sales territory management, resource budgeting.

  • Optimizes resource utilization
  • Enhances production efficiency
  • Maximizes profitability
  • Requires thorough analysis of past performance and market conditions
  • Potential for misallocation of resources if not executed properly

Frequently asked questions

How does the assignment method differ from other resource allocation methods, what factors should organizations consider when implementing the assignment method, can the assignment method be applied to non-profit organizations or public sector agencies, what role does technology play in implementing the assignment method, are there any ethical considerations associated with the assignment method, key takeaways.

  • The assignment method optimizes resource allocation to enhance efficiency and profitability.
  • Applications include workforce allocation, production planning, sales territory management, and resource budgeting.
  • Effective implementation requires thorough analysis of past performance and market conditions.
  • Strategic allocation of resources can drive overall performance and revenue growth.

Show Article Sources

You might also like.

what is the assignment methods

What is an Assignment Method?

Assignment Method

Share This...

Assignment method.

In accounting and finance, the assignment method is a technique used for allocating or assigning resources, costs, or tasks among different departments, employees, or projects. The assignment method aims to optimize resource allocation to achieve maximum efficiency, cost savings, or other desired outcomes. It is often used in cost accounting, project management, and operations research.

For example, in cost accounting , the assignment method can be used to allocate indirect costs (such as overhead) to various cost centers or cost objects based on certain allocation criteria, like the proportion of direct labor hours or machine hours. This helps in determining the total cost of each product or service and aids in decision-making related to pricing, production levels, or resource allocation.

Another example is in project management, where the assignment method can be used to allocate tasks to team members based on their skills, availability, or other relevant factors. This helps in efficient task distribution, ensuring timely project completion, and optimal utilization of resources.

In summary, the assignment method is a technique used for allocating resources, costs, or tasks to optimize efficiency and achieve desired outcomes.

Example of an Assignment Method

Let’s take an example from cost accounting , specifically in a manufacturing company.

Suppose a manufacturing company produces three products: Product A, Product B, and Product C. The company has a total indirect overhead cost of $90,000. The indirect overhead cost needs to be allocated to each product based on machine hours used in production.

The machine hours used for each product are as follows:

  • Product A: 600 hours
  • Product B: 900 hours
  • Product C: 1,500 hours

Total machine hours used: 3,000 hours (600 + 900 + 1,500)

Now, we will use the assignment method to allocate the indirect overhead costs based on the proportion of machine hours used for each product.

  • Calculate the overhead rate per machine hour: \(\text{Overhead rate} = \frac{\text{Total overhead cost}}{\text{Total machine hours}} \) \(\text{Overhead rate} = \frac{90,000}{3,000 \text{ hours}} \) Overhead rate = $30 per machine hour
  • Allocate the overhead cost to each product based on the machine hours used:
  • Product A: 600 hours * $30 = $18,000
  • Product B: 900 hours * $30 = $27,000
  • Product C: 1,500 hours * $30 = $45,000

So, using the assignment method, the allocated overhead costs for Product A, Product B, and Product C are $18,000, $27,000, and $45,000, respectively. This allocation helps the company understand the total cost of producing each product and make informed decisions about pricing, production levels, and resource allocation.

Other Posts You'll Like...

Understanding the Levels of the Judicial Process as it Related to Tax Matters

REG CPA Exam: Understanding the Levels of the Judicial Process as it Related to Tax Matters

Understanding the Tax Audit and Appeals Process

REG CPA Exam: Understanding the Tax Audit and Appeals Process

Understanding the Role and Authority of State Boards of Accountancy

REG CPA Exam: Understanding the Role and Authority of State Boards of Accountancy

Example Situations Resulting in Tax Return Preparer Penalties

REG CPA Exam: Example Situations Resulting in Tax Return Preparer Penalties

Understanding the Definition of a Tax Return Preparer or TRP

REG CPA Exam: Understanding the Definition of a Tax Return Preparer or TRP

Understanding the Regulations Governing Practice Before the IRS

REG CPA Exam: Understanding the Regulations Governing Practice Before the IRS

Helpful links.

  • Learn to Study "Strategically"
  • How to Pass a Failed CPA Exam
  • Samples of SFCPA Study Tools
  • SuperfastCPA Podcast

How Colbi Passed Her CPA Exams

“I Shouldn’t Be Able to Do This”: How Colbi Passed Her CPA Exams

5 cpa exam myths

The 5 Biggest Myths About CPA Exam Study

From 8 Hours a Day to 8 Hours a Week, How Branden Passed His CPA Exams

From 8 Hours a Day to 8 Hours a Week, How Branden Passed His CPA Exams

5 High Impact Study Strategies

5 High Impact CPA Study Strategies

How Melodie Passed Her CPA Exams by Making Every Morning Count

How Melodie Passed Her CPA Exams by Making Every Morning Count

Inconsistent CPA Study? Try These 4 Strategies

Inconsistent CPA Study? Try These 4 Strategies

Want to pass as fast as possible, ( and avoid failing sections ), watch one of our free "study hacks" trainings for a free walkthrough of the superfastcpa study methods that have helped so many candidates pass their sections faster and avoid failing scores....

what is the assignment methods

Make Your Study Process Easier and more effective with SuperfastCPA

Take Your CPA Exams with Confidence

  • Free "Study Hacks" Training
  • SuperfastCPA PRO Course
  • SuperfastCPA Review Notes
  • SuperfastCPA Audio Notes
  • SuperfastCPA Quizzes

Get Started

  • Free "Study Hacks Training"
  • Read Reviews of SuperfastCPA
  • Busy Candidate's Guide to Passing
  • Subscribe to the Podcast
  • Purchase Now
  • Nate's Story
  • Interviews with SFCPA Customers
  • Our Study Methods
  • SuperfastCPA Reviews
  • CPA Score Release Dates
  • The "Best" CPA Review Course
  • Do You Really Need the CPA License?
  • 7 Habits of Successful Candidates
  • "Deep Work" & CPA Study

The Writing Center • University of North Carolina at Chapel Hill

Understanding Assignments

What this handout is about.

The first step in any successful college writing venture is reading the assignment. While this sounds like a simple task, it can be a tough one. This handout will help you unravel your assignment and begin to craft an effective response. Much of the following advice will involve translating typical assignment terms and practices into meaningful clues to the type of writing your instructor expects. See our short video for more tips.

Basic beginnings

Regardless of the assignment, department, or instructor, adopting these two habits will serve you well :

  • Read the assignment carefully as soon as you receive it. Do not put this task off—reading the assignment at the beginning will save you time, stress, and problems later. An assignment can look pretty straightforward at first, particularly if the instructor has provided lots of information. That does not mean it will not take time and effort to complete; you may even have to learn a new skill to complete the assignment.
  • Ask the instructor about anything you do not understand. Do not hesitate to approach your instructor. Instructors would prefer to set you straight before you hand the paper in. That’s also when you will find their feedback most useful.

Assignment formats

Many assignments follow a basic format. Assignments often begin with an overview of the topic, include a central verb or verbs that describe the task, and offer some additional suggestions, questions, or prompts to get you started.

An Overview of Some Kind

The instructor might set the stage with some general discussion of the subject of the assignment, introduce the topic, or remind you of something pertinent that you have discussed in class. For example:

“Throughout history, gerbils have played a key role in politics,” or “In the last few weeks of class, we have focused on the evening wear of the housefly …”

The Task of the Assignment

Pay attention; this part tells you what to do when you write the paper. Look for the key verb or verbs in the sentence. Words like analyze, summarize, or compare direct you to think about your topic in a certain way. Also pay attention to words such as how, what, when, where, and why; these words guide your attention toward specific information. (See the section in this handout titled “Key Terms” for more information.)

“Analyze the effect that gerbils had on the Russian Revolution”, or “Suggest an interpretation of housefly undergarments that differs from Darwin’s.”

Additional Material to Think about

Here you will find some questions to use as springboards as you begin to think about the topic. Instructors usually include these questions as suggestions rather than requirements. Do not feel compelled to answer every question unless the instructor asks you to do so. Pay attention to the order of the questions. Sometimes they suggest the thinking process your instructor imagines you will need to follow to begin thinking about the topic.

“You may wish to consider the differing views held by Communist gerbils vs. Monarchist gerbils, or Can there be such a thing as ‘the housefly garment industry’ or is it just a home-based craft?”

These are the instructor’s comments about writing expectations:

“Be concise”, “Write effectively”, or “Argue furiously.”

Technical Details

These instructions usually indicate format rules or guidelines.

“Your paper must be typed in Palatino font on gray paper and must not exceed 600 pages. It is due on the anniversary of Mao Tse-tung’s death.”

The assignment’s parts may not appear in exactly this order, and each part may be very long or really short. Nonetheless, being aware of this standard pattern can help you understand what your instructor wants you to do.

Interpreting the assignment

Ask yourself a few basic questions as you read and jot down the answers on the assignment sheet:

Why did your instructor ask you to do this particular task?

Who is your audience.

  • What kind of evidence do you need to support your ideas?

What kind of writing style is acceptable?

  • What are the absolute rules of the paper?

Try to look at the question from the point of view of the instructor. Recognize that your instructor has a reason for giving you this assignment and for giving it to you at a particular point in the semester. In every assignment, the instructor has a challenge for you. This challenge could be anything from demonstrating an ability to think clearly to demonstrating an ability to use the library. See the assignment not as a vague suggestion of what to do but as an opportunity to show that you can handle the course material as directed. Paper assignments give you more than a topic to discuss—they ask you to do something with the topic. Keep reminding yourself of that. Be careful to avoid the other extreme as well: do not read more into the assignment than what is there.

Of course, your instructor has given you an assignment so that they will be able to assess your understanding of the course material and give you an appropriate grade. But there is more to it than that. Your instructor has tried to design a learning experience of some kind. Your instructor wants you to think about something in a particular way for a particular reason. If you read the course description at the beginning of your syllabus, review the assigned readings, and consider the assignment itself, you may begin to see the plan, purpose, or approach to the subject matter that your instructor has created for you. If you still aren’t sure of the assignment’s goals, try asking the instructor. For help with this, see our handout on getting feedback .

Given your instructor’s efforts, it helps to answer the question: What is my purpose in completing this assignment? Is it to gather research from a variety of outside sources and present a coherent picture? Is it to take material I have been learning in class and apply it to a new situation? Is it to prove a point one way or another? Key words from the assignment can help you figure this out. Look for key terms in the form of active verbs that tell you what to do.

Key Terms: Finding Those Active Verbs

Here are some common key words and definitions to help you think about assignment terms:

Information words Ask you to demonstrate what you know about the subject, such as who, what, when, where, how, and why.

  • define —give the subject’s meaning (according to someone or something). Sometimes you have to give more than one view on the subject’s meaning
  • describe —provide details about the subject by answering question words (such as who, what, when, where, how, and why); you might also give details related to the five senses (what you see, hear, feel, taste, and smell)
  • explain —give reasons why or examples of how something happened
  • illustrate —give descriptive examples of the subject and show how each is connected with the subject
  • summarize —briefly list the important ideas you learned about the subject
  • trace —outline how something has changed or developed from an earlier time to its current form
  • research —gather material from outside sources about the subject, often with the implication or requirement that you will analyze what you have found

Relation words Ask you to demonstrate how things are connected.

  • compare —show how two or more things are similar (and, sometimes, different)
  • contrast —show how two or more things are dissimilar
  • apply—use details that you’ve been given to demonstrate how an idea, theory, or concept works in a particular situation
  • cause —show how one event or series of events made something else happen
  • relate —show or describe the connections between things

Interpretation words Ask you to defend ideas of your own about the subject. Do not see these words as requesting opinion alone (unless the assignment specifically says so), but as requiring opinion that is supported by concrete evidence. Remember examples, principles, definitions, or concepts from class or research and use them in your interpretation.

  • assess —summarize your opinion of the subject and measure it against something
  • prove, justify —give reasons or examples to demonstrate how or why something is the truth
  • evaluate, respond —state your opinion of the subject as good, bad, or some combination of the two, with examples and reasons
  • support —give reasons or evidence for something you believe (be sure to state clearly what it is that you believe)
  • synthesize —put two or more things together that have not been put together in class or in your readings before; do not just summarize one and then the other and say that they are similar or different—you must provide a reason for putting them together that runs all the way through the paper
  • analyze —determine how individual parts create or relate to the whole, figure out how something works, what it might mean, or why it is important
  • argue —take a side and defend it with evidence against the other side

More Clues to Your Purpose As you read the assignment, think about what the teacher does in class:

  • What kinds of textbooks or coursepack did your instructor choose for the course—ones that provide background information, explain theories or perspectives, or argue a point of view?
  • In lecture, does your instructor ask your opinion, try to prove their point of view, or use keywords that show up again in the assignment?
  • What kinds of assignments are typical in this discipline? Social science classes often expect more research. Humanities classes thrive on interpretation and analysis.
  • How do the assignments, readings, and lectures work together in the course? Instructors spend time designing courses, sometimes even arguing with their peers about the most effective course materials. Figuring out the overall design to the course will help you understand what each assignment is meant to achieve.

Now, what about your reader? Most undergraduates think of their audience as the instructor. True, your instructor is a good person to keep in mind as you write. But for the purposes of a good paper, think of your audience as someone like your roommate: smart enough to understand a clear, logical argument, but not someone who already knows exactly what is going on in your particular paper. Remember, even if the instructor knows everything there is to know about your paper topic, they still have to read your paper and assess your understanding. In other words, teach the material to your reader.

Aiming a paper at your audience happens in two ways: you make decisions about the tone and the level of information you want to convey.

  • Tone means the “voice” of your paper. Should you be chatty, formal, or objective? Usually you will find some happy medium—you do not want to alienate your reader by sounding condescending or superior, but you do not want to, um, like, totally wig on the man, you know? Eschew ostentatious erudition: some students think the way to sound academic is to use big words. Be careful—you can sound ridiculous, especially if you use the wrong big words.
  • The level of information you use depends on who you think your audience is. If you imagine your audience as your instructor and they already know everything you have to say, you may find yourself leaving out key information that can cause your argument to be unconvincing and illogical. But you do not have to explain every single word or issue. If you are telling your roommate what happened on your favorite science fiction TV show last night, you do not say, “First a dark-haired white man of average height, wearing a suit and carrying a flashlight, walked into the room. Then a purple alien with fifteen arms and at least three eyes turned around. Then the man smiled slightly. In the background, you could hear a clock ticking. The room was fairly dark and had at least two windows that I saw.” You also do not say, “This guy found some aliens. The end.” Find some balance of useful details that support your main point.

You’ll find a much more detailed discussion of these concepts in our handout on audience .

The Grim Truth

With a few exceptions (including some lab and ethnography reports), you are probably being asked to make an argument. You must convince your audience. It is easy to forget this aim when you are researching and writing; as you become involved in your subject matter, you may become enmeshed in the details and focus on learning or simply telling the information you have found. You need to do more than just repeat what you have read. Your writing should have a point, and you should be able to say it in a sentence. Sometimes instructors call this sentence a “thesis” or a “claim.”

So, if your instructor tells you to write about some aspect of oral hygiene, you do not want to just list: “First, you brush your teeth with a soft brush and some peanut butter. Then, you floss with unwaxed, bologna-flavored string. Finally, gargle with bourbon.” Instead, you could say, “Of all the oral cleaning methods, sandblasting removes the most plaque. Therefore it should be recommended by the American Dental Association.” Or, “From an aesthetic perspective, moldy teeth can be quite charming. However, their joys are short-lived.”

Convincing the reader of your argument is the goal of academic writing. It doesn’t have to say “argument” anywhere in the assignment for you to need one. Look at the assignment and think about what kind of argument you could make about it instead of just seeing it as a checklist of information you have to present. For help with understanding the role of argument in academic writing, see our handout on argument .

What kind of evidence do you need?

There are many kinds of evidence, and what type of evidence will work for your assignment can depend on several factors–the discipline, the parameters of the assignment, and your instructor’s preference. Should you use statistics? Historical examples? Do you need to conduct your own experiment? Can you rely on personal experience? See our handout on evidence for suggestions on how to use evidence appropriately.

Make sure you are clear about this part of the assignment, because your use of evidence will be crucial in writing a successful paper. You are not just learning how to argue; you are learning how to argue with specific types of materials and ideas. Ask your instructor what counts as acceptable evidence. You can also ask a librarian for help. No matter what kind of evidence you use, be sure to cite it correctly—see the UNC Libraries citation tutorial .

You cannot always tell from the assignment just what sort of writing style your instructor expects. The instructor may be really laid back in class but still expect you to sound formal in writing. Or the instructor may be fairly formal in class and ask you to write a reflection paper where you need to use “I” and speak from your own experience.

Try to avoid false associations of a particular field with a style (“art historians like wacky creativity,” or “political scientists are boring and just give facts”) and look instead to the types of readings you have been given in class. No one expects you to write like Plato—just use the readings as a guide for what is standard or preferable to your instructor. When in doubt, ask your instructor about the level of formality they expect.

No matter what field you are writing for or what facts you are including, if you do not write so that your reader can understand your main idea, you have wasted your time. So make clarity your main goal. For specific help with style, see our handout on style .

Technical details about the assignment

The technical information you are given in an assignment always seems like the easy part. This section can actually give you lots of little hints about approaching the task. Find out if elements such as page length and citation format (see the UNC Libraries citation tutorial ) are negotiable. Some professors do not have strong preferences as long as you are consistent and fully answer the assignment. Some professors are very specific and will deduct big points for deviations.

Usually, the page length tells you something important: The instructor thinks the size of the paper is appropriate to the assignment’s parameters. In plain English, your instructor is telling you how many pages it should take for you to answer the question as fully as you are expected to. So if an assignment is two pages long, you cannot pad your paper with examples or reword your main idea several times. Hit your one point early, defend it with the clearest example, and finish quickly. If an assignment is ten pages long, you can be more complex in your main points and examples—and if you can only produce five pages for that assignment, you need to see someone for help—as soon as possible.

Tricks that don’t work

Your instructors are not fooled when you:

  • spend more time on the cover page than the essay —graphics, cool binders, and cute titles are no replacement for a well-written paper.
  • use huge fonts, wide margins, or extra spacing to pad the page length —these tricks are immediately obvious to the eye. Most instructors use the same word processor you do. They know what’s possible. Such tactics are especially damning when the instructor has a stack of 60 papers to grade and yours is the only one that low-flying airplane pilots could read.
  • use a paper from another class that covered “sort of similar” material . Again, the instructor has a particular task for you to fulfill in the assignment that usually relates to course material and lectures. Your other paper may not cover this material, and turning in the same paper for more than one course may constitute an Honor Code violation . Ask the instructor—it can’t hurt.
  • get all wacky and “creative” before you answer the question . Showing that you are able to think beyond the boundaries of a simple assignment can be good, but you must do what the assignment calls for first. Again, check with your instructor. A humorous tone can be refreshing for someone grading a stack of papers, but it will not get you a good grade if you have not fulfilled the task.

Critical reading of assignments leads to skills in other types of reading and writing. If you get good at figuring out what the real goals of assignments are, you are going to be better at understanding the goals of all of your classes and fields of study.

You may reproduce it for non-commercial use if you use the entire handout and attribute the source: The Writing Center, University of North Carolina at Chapel Hill

Make a Gift

Cost Allocation

The process of identifying, accumulating, and assigning costs to costs objects

What is Cost Allocation?

Cost allocation is the process of identifying, accumulating, and assigning costs to costs objects such as departments, products, programs, or a branch of a company. It involves identifying the cost objects in a company, identifying the costs incurred by the cost objects, and then assigning the costs to the cost objects based on specific criteria.

Cost Allocation Diagram - How It Works

When costs are allocated in the right way, the business is able to trace the specific cost objects that are making profits or losses for the company. If costs are allocated to the wrong cost objects, the company may be assigning resources to cost objects that do not yield as much profits as expected.

Types of Costs

There are several types of costs that an organization must define before allocating costs to their specific cost objects. These costs include:

1. Direct costs

Direct costs are costs that can be attributed to a specific product or service, and they do not need to be allocated to the specific cost object. It is because the organization knows what expenses go to the specific departments that generate profits and the costs incurred in producing specific products or services . For example, the salaries paid to factory workers assigned to a specific division is known and does not need to be allocated again to that division.

2. Indirect costs

Indirect costs are costs that are not directly related to a specific cost object like a function, product, or department. They are costs that are needed for the sake of the company’s operations and health. Some common examples of indirect costs include security costs, administration costs, etc. The costs are first identified, pooled, and then allocated to specific cost objects within the organization.

Indirect costs can be divided into fixed and variable costs. Fixed costs are costs that are fixed for a specific product or department. An example of a fixed cost is the remuneration of a project supervisor assigned to a specific division. The other category of indirect cost is variable costs, which vary with the level of output. Indirect costs increase or decrease with changes in the level of output.

3. Overhead costs

Overhead costs are indirect costs that are not part of manufacturing costs. They are not related to the labor or material costs that are incurred in the production of goods or services. They support the production or selling processes of the goods or services. Overhead costs are charged to the expense account, and they must be continually paid regardless of whether the company is selling goods or not.

Some common examples of overhead costs are rental expenses, utilities, insurance, postage and printing, administrative and legal expenses , and research and development costs.

Cost Allocation Mechanism

The following are the main steps involved when allocating costs to cost objects:

1. Identify cost objects

The first step when allocating costs is to identify the cost objects for which the organization needs to separately estimate the associated cost. Identifying specific cost objects is important because they are the drivers of the business, and decisions are made with them in mind.

The cost object can be a brand , project, product line, division/department, or a branch of the company. The company should also determine the cost allocation base, which is the basis that it uses to allocate the costs to cost objects.

2. Accumulate costs into a cost pool

After identifying the cost objects, the next step is to accumulate the costs into a cost pool, pending allocation to the cost objects. When accumulating costs, you can create several categories where the costs will be pooled based on the cost allocation base used. Some examples of cost pools include electricity usage, water usage, square footage, insurance, rent expenses , fuel consumption, and motor vehicle maintenance.

What is a Cost Driver?

A cost driver causes a change in the cost associated with an activity. Some examples of cost drivers include the number of machine-hours, the number of direct labor hours worked, the number of payments processed, the number of purchase orders, and the number of invoices sent to customers.

Benefits of Cost Allocation

The following are some of the reasons why cost allocation is important to an organization:

1. Assists in the decision-making process

Cost allocation provides the management with important data about cost utilization that they can use in making decisions. It shows the cost objects that take up most of the costs and helps determine if the departments or products are profitable enough to justify the costs allocated. For unprofitable cost objects, the company’s management can cut the costs allocated and divert the money to other more profitable cost objects.

2. Helps evaluate and motivate staff

Cost allocation helps determine if specific departments are profitable or not. If the cost object is not profitable, the company can evaluate the performance of the staff members to determine if a decline in productivity is the cause of the non-profitability of the cost objects.

On the other hand, if the company recognizes and rewards a specific department for achieving the highest profitability in the company, the employees assigned to that department will be motivated to work hard and continue with their good performance.

Additional Resources

Thank you for reading CFI’s guide to Cost Allocation. In order to help you become a world-class financial analyst and advance your career to your fullest potential, these additional resources will be very helpful:

  • Break-Even Analysis
  • Cost of Production
  • Fixed and Variable Costs
  • Projecting Income Statement Line Items
  • See all accounting resources

what is the assignment methods

  • Share this article

Excel Fundamentals - Formulas for Finance

Create a free account to unlock this Template

Access and download collection of free Templates to help power your productivity and performance.

Already have an account? Log in

Supercharge your skills with Premium Templates

Take your learning and productivity to the next level with our Premium Templates.

Upgrading to a paid membership gives you access to our extensive collection of plug-and-play Templates designed to power your performance—as well as CFI's full course catalog and accredited Certification Programs.

Already have a Self-Study or Full-Immersion membership? Log in

Access Exclusive Templates

Gain unlimited access to more than 250 productivity Templates, CFI's full course catalog and accredited Certification Programs, hundreds of resources, expert reviews and support, the chance to work with real-world finance and research tools, and more.

Already have a Full-Immersion membership? Log in

  • Investing 101
  • The 4 Best S&P 500 Index Funds
  • World's Top 20 Economies
  • Stock Basics Tutorial
  • Options Basics Tutorial
  • Economics Basics
  • Mutual Funds
  • Bonds/Fixed Income
  • Commodities
  • Company News
  • Market/Economy News
  • Apple (AAPL)
  • Tesla (TSLA)
  • Amazon (AMZN)
  • Facebook (FB)
  • Netflix (NFLX)
  • Create an Account
  • Join a Game
  • Create a Game
  • Budgeting/Saving
  • Small Business
  • Wealth Management
  • Broker Reviews
  • Charles Schwab Review
  • E*TRADE Review
  • Robinhood Review
  • Advisor Insights
  • Investopedia 100
  • Join Advisor Insights
  • Your Practice
  • Investing for Beginners
  • Become a Day Trader
  • Trading for Beginners
  • Technical Analysis
  • All Courses
  • Trading Courses
  • Investing Courses
  • Financial Professional Courses
  • Advisor Login
  • Newsletters

Financial Analysis

  • Investing Strategy
  • Bonds / Fixed Income
  • Real Estate
  • International / Global
  • Alternative Investments
  • Asset Allocation
  • Sustainable Investing

Assignment Method

What is assignment method.

Assignment method is a way of allocating organizational resources where a resource is assigned to a particular task. The resource would be monetary, personnel, technological or another type of resource. The assignment method is used to determine what resources are assigned to which department, machine, or center of operation in the production process. This method is used to allocate the proper number of employees to a machine or task, and the number of jobs that a given machine or factory can produce. The idea is to assign resources in such a way that profits are maximized.

BREAKING DOWN Assignment Method

The assignment method is a way of allocating organizational resources to projects and tasks. The assignment method can be used for many other purposes besides production allocations. It can be employed to assign the number of salespersons to a given territory or territories. It can also be used to match bidders to contracts and assign other relevant components of business to each other. Regardless of the resource being allocated or the task to be accomplished, the idea is to assign resources in such a way that maximizes the amount of profit produced by the task or project.

Related Terms

Related articles, how do the equity method and proportional consolidation method differ, what are the main methods for calculating business costs, how to use the bayesian method of financial forecasting, cost accounting method: advantages and disadvantages, understanding methods and assumptions of depreciation, the differences between the installment method and percentage of completion method.

  • Privacy Policy
  • Terms of Use

Web Analytics

Study.com

In order to continue enjoying our site, we ask that you confirm your identity as a human. Thank you very much for your cooperation.

Assignment Problem: Meaning, Methods and Variations | Operations Research

what is the assignment methods

After reading this article you will learn about:- 1. Meaning of Assignment Problem 2. Definition of Assignment Problem 3. Mathematical Formulation 4. Hungarian Method 5. Variations.

Meaning of Assignment Problem:

An assignment problem is a particular case of transportation problem where the objective is to assign a number of resources to an equal number of activities so as to minimise total cost or maximize total profit of allocation.

The problem of assignment arises because available resources such as men, machines etc. have varying degrees of efficiency for performing different activities, therefore, cost, profit or loss of performing the different activities is different.

Thus, the problem is “How should the assignments be made so as to optimize the given objective”. Some of the problem where the assignment technique may be useful are assignment of workers to machines, salesman to different sales areas.

Definition of Assignment Problem:

ADVERTISEMENTS:

Suppose there are n jobs to be performed and n persons are available for doing these jobs. Assume that each person can do each job at a term, though with varying degree of efficiency, let c ij be the cost if the i-th person is assigned to the j-th job. The problem is to find an assignment (which job should be assigned to which person one on-one basis) So that the total cost of performing all jobs is minimum, problem of this kind are known as assignment problem.

The assignment problem can be stated in the form of n x n cost matrix C real members as given in the following table:

what is the assignment methods

Advantages and Disadvantages of Assignment Method

Looking for advantages and disadvantages of Assignment Method?

We have collected some solid points that will help you understand the pros and cons of Assignment Method in detail.

But first, let’s understand the topic:

What is Assignment Method?

What are the advantages and disadvantages of assignment method.

The following are the advantages and disadvantages of Assignment Method:

AdvantagesDisadvantages
Promotes efficient task allocationCan be time-consuming
Enhances productivityLimited to quantitative tasks
Encourages skills developmentIgnores individual preferences
Minimizes project completion timeMay not promote creativity
Boosts employee job satisfactionRisk of biased assignments

Advantages and disadvantages of Assignment Method

Advantages of Assignment Method

Disadvantages of assignment method.

You can view other “advantages and disadvantages of…” posts by clicking here .

If you have a related query, feel free to let us know in the comments below.

Also, kindly share the information with your friends who you think might be interested in reading it.

Leave a Reply Cancel reply

Save my name, email, and website in this browser for the next time I comment.

what is the assignment methods

Types of Assignments and Assessments

Assignments and assessments are much the same thing: an instructor is unlikely to give students an assignment that does not receive some sort of assessment, whether formal or informal, formative or summative; and an assessment must be assigned, whether it is an essay, case study, or final exam. When the two terms are distinquished, "assignment" tends to refer to a learning activity that is primarily intended to foster or consolidate learning, while "assessment" tends to refer to an activity that is primarily intended to measure how well a student has learned. 

In the list below, some attempt has been made to put the assignments/assessments in into logical categories. However, many of them could appear in multiple categories, so to prevent the list from becoming needlessly long, each item has been allocated to just one category. 

Written Assignments:

  • Annotated Bibliography : An annotated bibliography is a list of citations or references to sources such as books, articles, websites, etc., along with brief descriptions or annotations that summarize, evaluate, and explain the content, relevance, and quality of each source. These annotations provide readers with insights into the source's content and its potential usefulness for research or reference.
  • Summary/Abstract : A summary or abstract is a concise and condensed version of a longer document or research article, presenting the main points, key findings, and essential information in a clear and brief manner. It allows readers to quickly grasp the main ideas and determine whether the full document is relevant to their needs or interests. Abstracts are commonly found at the beginning of academic papers, research articles, and reports, providing a snapshot of the entire content.
  • Case Analysis : Case analysis refers to a systematic examination and evaluation of a particular situation, problem, or scenario. It involves gathering relevant information, identifying key factors, analyzing various aspects, and formulating conclusions or recommendations based on the findings. Case analysis is commonly used in business, law, and other fields to make informed decisions and solve complex problems.
  • Definition : A definition is a clear and concise explanation that describes the meaning of a specific term, concept, or object. It aims to provide a precise understanding of the item being defined, often by using words, phrases, or context that distinguish it from other similar or related things.
  • Description of a Process : A description of a process is a step-by-step account or narrative that outlines the sequence of actions, tasks, or events involved in completing a particular activity or achieving a specific goal. Process descriptions are commonly used in various industries to document procedures, guide employees, and ensure consistent and efficient workflows.
  • Executive Summary : An executive summary is a condensed version of a longer document or report that provides an overview of the main points, key findings, and major recommendations. It is typically aimed at busy executives or decision-makers who need a quick understanding of the content without delving into the full details. Executive summaries are commonly used in business proposals, project reports, and research papers to present essential information concisely.
  • Proposal/Plan : A piece of writing that explains how a future problem or project will be approached.
  • Laboratory or Field Notes:  Laboratory/field notes are detailed and systematic written records taken by scientists, researchers, or students during experiments, observations, or fieldwork. These notes document the procedures, observations, data, and any unexpected findings encountered during the scientific investigation. They serve as a vital reference for later analysis, replication, and communication of the research process and results.
  • Research Paper : A research paper is a more extensive and in-depth academic work that involves original research, data collection from multiple sources, and analysis. It aims to contribute new insights to the existing body of knowledge on a specific subject. Compare to "essay" below.
  • Essay : A composition that calls for exposition of a thesis and is composed of several paragraphs including an introduction, a body, and a conclusion. It is different from a research paper in that the synthesis of bibliographic sources is not required. Compare to "Research Paper" above. 
  • Memo : A memo, short for memorandum, is a brief written message or communication used within an organization or business. It is often used to convey information, provide updates, make announcements, or request actions from colleagues or team members.
  • Micro-theme : A micro-theme refers to a concise and focused piece of writing that addresses a specific topic or question. It is usually shorter than a traditional essay or research paper and requires the writer to present their ideas clearly and concisely.
  • Notes on Reading : Notes on reading are annotations, comments, or summaries taken while reading a book, article, or any other written material. They serve as aids for understanding, retention, and later reference, helping the reader recall essential points and ideas from the text.
  • Outline : An outline is a structured and organized plan that lays out the main points and structure of a written work, such as an essay, research paper, or presentation. It provides a roadmap for the writer, ensuring logical flow and coherence in the final piece.
  • Plan for Conducting a Project : A plan for conducting a project outlines the steps, resources, timelines, and objectives for successfully completing a specific project. It includes details on how tasks will be executed and managed to achieve the desired outcomes.
  • Poem : A poem is a literary work written in verse, using poetic devices like rhythm, rhyme, and imagery to convey emotions, ideas, and experiences.
  • Play : A play is a form of literature written for performance, typically involving dialogue and actions by characters to tell a story or convey a message on stage.
  • Choreography : Choreography refers to the art of designing dance sequences or movements, often for performances in various dance styles.
  • Article/Book Review : An article or book review is a critical evaluation and analysis of a piece of writing, such as an article or a book. It typically includes a summary of the content and the reviewer's assessment of its strengths, weaknesses, and overall value.
  • Review of Literature : A review of literature is a comprehensive summary and analysis of existing research and scholarly writings on a particular topic. It aims to provide an overview of the current state of knowledge in a specific field and may be a part of academic research or a standalone piece.
  • Essay-based Exam : An essay-based exam is an assessment format where students are required to respond to questions or prompts with written, structured responses. It involves expressing ideas, arguments, and explanations in a coherent and organized manner, often requiring critical thinking and analysis.
  • "Start" : In the context of academic writing, "start" refers to the initial phase of organizing and planning a piece of writing. It involves formulating a clear and focused thesis statement, which presents the main argument or central idea of the work, and creating an outline or list of ideas that will support and develop the thesis throughout the writing process.
  • Statement of Assumptions : A statement of assumptions is a declaration or acknowledgment made at the beginning of a document or research paper, highlighting the underlying beliefs, conditions, or premises on which the work is based. It helps readers understand the foundation of the writer's perspective and the context in which the content is presented.
  • Summary or Precis : A summary or precis is a concise and condensed version of a longer piece of writing, such as an article, book, or research paper. It captures the main points, key arguments, and essential information in a succinct manner, enabling readers to grasp the content without reading the full text.
  • Unstructured Writing : Unstructured writing refers to the process of writing without following a specific plan, outline, or organizational structure. It allows the writer to freely explore ideas, thoughts, and creativity without the constraints of a predefined format or order. Unstructured writing is often used for brainstorming, creative expression, or personal reflection.
  • Rough Draft or Freewrite : A rough draft or freewrite is an initial version of a piece of writing that is not polished or edited. It serves as an early attempt by the writer to get ideas on paper without worrying about perfection, allowing for exploration and creativity before revising and refining the final version.
  • Technical or Scientific Report : A technical or scientific report is a document that presents detailed information about a specific technical or scientific project, research study, experiment, or investigation. It follows a structured format and includes sections like abstract, introduction, methods, results, discussion, and conclusion to communicate findings and insights in a clear and systematic manner.
  • Journal article : A formal article reporting original research that could be submitted to an academic journal. Rather than a format dictated by the professor, the writer must use the conventional form of academic journals in the relevant discipline.
  • Thesis statement : A clear and concise sentence or two that presents the main argument or central claim of an essay, research paper, or any written piece. It serves as a roadmap for the reader, outlining the writer's stance on the topic and the key points that will be discussed and supported in the rest of the work. The thesis statement provides focus and direction to the paper, guiding the writer's approach to the subject matter and helping to maintain coherence throughout the writing.

Visual Representation

  • Brochure : A brochure is a printed or digital document used for advertising, providing information, or promoting a product, service, or event. It typically contains a combination of text and visuals, such as images or graphics, arranged in a visually appealing layout to convey a message effectively.
  • Poster : A poster is a large printed visual display intended to catch the attention of an audience. It often contains a combination of text, images, and graphics to communicate information or promote a particular message, event, or cause.
  • Chart : A chart is a visual representation of data or information using various formats such as pie charts, bar charts, line charts, or tables. It helps to illustrate relationships, trends, and comparisons in a concise and easy-to-understand manner.
  • Graph : A graph is a visual representation of numerical data, usually presented using lines, bars, points, or other symbols on a coordinate plane. Graphs are commonly used to show trends, patterns, and relationships between variables.
  • Concept Map : A concept map is a graphical tool used to organize and represent the connections and relationships between different concepts or ideas. It typically uses nodes or boxes to represent concepts and lines or arrows to show the connections or links between them, helping to visualize the relationships and hierarchy of ideas.
  • Diagram : A diagram is a visual representation of a process, system, or structure using labeled symbols, shapes, or lines. Diagrams are used to explain complex concepts or procedures in a simplified and easy-to-understand manner.
  • Table : A table is a systematic arrangement of data or information in rows and columns, allowing for easy comparison and reference. It is commonly used to present numerical data or detailed information in an organized format.
  • Flowchart : A flowchart is a graphical representation of a process, workflow, or algorithm, using various shapes and arrows to show the sequence of steps or decisions involved. It helps visualize the logical flow and decision points, making it easier to understand and analyze complex processes.
  • Multimedia or Slide Presentation : A multimedia or slide presentation is a visual communication tool that combines text, images, audio, video, and other media elements to deliver information or a message to an audience. It is often used for educational, business, or informational purposes and can be presented in person or virtually using software like Microsoft PowerPoint or Google Slides.
  • ePortfolio : An ePortfolio, short for electronic portfolio, is a digital collection of an individual's work, accomplishments, skills, and reflections. It typically includes a variety of multimedia artifacts such as documents, presentations, videos, images, and links to showcase a person's academic, professional, or personal achievements. Eportfolios are used for self-reflection, professional development, and showcasing one's abilities to potential employers, educators, or peers. They provide a comprehensive and organized way to present evidence of learning, growth, and accomplishments over time.

Multiple-Choice Questions : These questions present a statement or question with several possible answer options, of which one or more may be correct. Test-takers must select the most appropriate choice(s). See CTE's Teaching Tip "Designing Multiple-Choice Questions."  

True or False Questions : These questions require test-takers to determine whether a given statement is true or false based on their knowledge of the subject.

Short-Answer Questions : Test-takers are asked to provide brief written responses to questions or prompts. These responses are usually a few sentences or a paragraph in length.

Essay Questions : Essay questions require test-takers to provide longer, more detailed written responses to a specific topic or question. They may involve analysis, critical thinking, and the development of coherent arguments.

Matching Questions : In matching questions, test-takers are asked to pair related items from two lists. They must correctly match the items based on their associations.

Fill-in-the-Blank Questions : Test-takers must complete sentences or passages by filling in the missing words or phrases. This type of question tests recall and understanding of specific information.

Multiple-Response Questions : Similar to multiple-choice questions, but with multiple correct options. Test-takers must select all the correct choices to receive full credit.

Diagram or Image-Based Questions : These questions require test-takers to analyze or interpret diagrams, charts, graphs, or images to answer specific queries.

Problem-Solving Questions : These questions present real-world or theoretical problems that require test-takers to apply their knowledge and skills to arrive at a solution.

Vignettes or Case-Based Questions : In these questions, test-takers are presented with a scenario or case study and must analyze the information to answer related questions.

Sequencing or Order Questions : Test-takers are asked to arrange items or events in a particular order or sequence based on their understanding of the subject matter.

Projects intended for a specific audience :

  • Advertisement : An advertisement is a promotional message or communication aimed at promoting a product, service, event, or idea to a target audience. It often uses persuasive techniques, visuals, and compelling language to attract attention and encourage consumers to take specific actions, such as making a purchase or seeking more information.
  • Client Report for an Agency : A client report for an agency is a formal document prepared by a service provider or agency to communicate the results, progress, or recommendations of their work to their client. It typically includes an analysis of data, achievements, challenges, and future plans related to the project or services provided.
  • News or Feature Story : A news story is a journalistic piece that reports on current events or recent developments, providing objective information in a factual and unbiased manner. A feature story, on the other hand, is a more in-depth and creative piece that explores human interest topics, profiles individuals, or delves into issues from a unique perspective.
  • Instructional Manual : An instructional manual is a detailed document that provides step-by-step guidance, explanations, and procedures on how to use, assemble, operate, or perform specific tasks with a product or system. It aims to help users understand and utilize the item effectively and safely.
  • Letter to the Editor : A letter to the editor is a written communication submitted by a reader to a newspaper, magazine, or online publication, expressing their opinion, feedback, or comments on a particular article, topic, or issue. It is intended for publication and allows individuals to share their perspectives with a broader audience.

Problem-Solving and Analysis :

  • Taxonomy : Taxonomy is the science of classification, categorization, and naming of organisms, objects, or concepts based on their characteristics, similarities, and differences. It involves creating hierarchical systems that group related items together, facilitating organization and understanding within a particular domain.
  • Budget with Rationale : A budget with rationale is a financial plan that outlines projected income and expenses for a specific period, such as a month or a year. The rationale provides explanations or justifications for each budget item, explaining the purpose and reasoning behind the allocated funds.
  • Case Analysis : Case analysis refers to a methodical examination of a particular situation, scenario, or problem. It involves gathering relevant data, identifying key issues, analyzing different factors, and formulating conclusions or recommendations based on the findings. Case analysis is commonly used in various fields, such as business, law, and education, to make informed decisions and solve complex problems.
  • Case Study : A case study is an in-depth analysis of a specific individual, group, organization, or situation. It involves thorough research, data collection, and detailed examination to understand the context, challenges, and outcomes associated with the subject of study. Case studies are widely used in academic research and professional contexts to gain insights into real-world scenarios.
  • Word Problem : A word problem is a type of mathematical or logical question presented in a contextual format using words rather than purely numerical or symbolic representations. It challenges students to apply their knowledge and problem-solving skills to real-life situations.

Collaborative Activities

  • Debate : A debate is a structured discussion between two or more individuals or teams with differing viewpoints on a specific topic or issue. Participants present arguments and counterarguments to support their positions, aiming to persuade the audience and ultimately reach a resolution or conclusion. Debates are commonly used in academic settings, public forums, and formal competitions to foster critical thinking, communication skills, and understanding of diverse perspectives.
  • Group Discussion : A group discussion is an interactive conversation involving several individuals who come together to exchange ideas, opinions, and information on a particular subject. The discussion is typically moderated to ensure that everyone has an opportunity to participate, and it encourages active listening, collaboration, and problem-solving. Group discussions are commonly used in educational settings, team meetings, and decision-making processes to promote dialogue and collective decision-making.
  • An oral report is a form of communication in which a person or group of persons present information, findings, or ideas verbally to an audience. It involves speaking in front of others, often in a formal setting, and delivering a structured presentation that may include visual aids, such as slides or props, to support the content. Oral reports are commonly used in academic settings, business environments, and various professional settings to share knowledge, research findings, project updates, or persuasive arguments. Effective oral reports require clear organization, articulation, and engaging delivery to effectively convey the intended message to the listeners.

Planning and Organization

  • Inventory : An inventory involves systematically listing and categorizing items or resources to assess their availability, quantity, and condition. In an educational context, students might conduct an inventory of books in a library, equipment in a lab, or supplies in a classroom, enhancing their organizational and data collection skills.
  • Materials and Methods Plan : A materials and methods plan involves developing a structured outline or description of the materials, tools, and procedures to be used in a specific experiment, research project, or practical task. It helps learners understand the importance of proper planning and documentation in scientific and research endeavors.
  • Plan for Conducting a Project : This learning activity requires students to create a detailed roadmap for executing a project. It includes defining the project's objectives, identifying tasks and timelines, allocating resources, and setting milestones to monitor progress. It enhances students' project management and organizational abilities.
  • Research Proposal Addressed to a Granting Agency : A formal document requesting financial support for a research project from a granting agency or organization. The proposal outlines the research questions, objectives, methodology, budget, and potential outcomes. It familiarizes learners with the process of seeking funding and strengthens their research and persuasive writing skills.
  • Mathematical Problem : A mathematical problem is a task or question that requires the application of mathematical principles, formulas, or operations to find a solution. It could involve arithmetic, algebra, geometry, calculus, or other branches of mathematics, challenging individuals to solve the problem logically and accurately.
  • Question : A question is a sentence or phrase used to elicit information, seek clarification, or provoke thought from someone else. Questions can be open-ended, closed-ended, or leading, depending on their purpose, and they play a crucial role in communication, problem-solving, and learning.

More Resources

CTE Teaching Tips

  • Personal Response Systems
  • Designing Multiple-Choice Questions
  • Aligning Outcomes, Assessments, and Instruction

Other Resources

  • Types of Assignments . University of Queensland.

If you would like support applying these tips to your own teaching, CTE staff members are here to help.  View the  CTE Support  page to find the most relevant staff member to contact.

teachingitps

Catalog search

Teaching tip categories.

  • Assessment and feedback
  • Blended Learning and Educational Technologies
  • Career Development
  • Course Design
  • Course Implementation
  • Inclusive Teaching and Learning
  • Learning activities
  • Support for Student Learning
  • Support for TAs
  • Assessment and feedback ,
  • Privacy Policy

Research Method

Home » Assignment – Types, Examples and Writing Guide

Assignment – Types, Examples and Writing Guide

Table of Contents

Assignment

Definition:

Assignment is a task given to students by a teacher or professor, usually as a means of assessing their understanding and application of course material. Assignments can take various forms, including essays, research papers, presentations, problem sets, lab reports, and more.

Assignments are typically designed to be completed outside of class time and may require independent research, critical thinking, and analysis. They are often graded and used as a significant component of a student’s overall course grade. The instructions for an assignment usually specify the goals, requirements, and deadlines for completion, and students are expected to meet these criteria to earn a good grade.

History of Assignment

The use of assignments as a tool for teaching and learning has been a part of education for centuries. Following is a brief history of the Assignment.

  • Ancient Times: Assignments such as writing exercises, recitations, and memorization tasks were used to reinforce learning.
  • Medieval Period : Universities began to develop the concept of the assignment, with students completing essays, commentaries, and translations to demonstrate their knowledge and understanding of the subject matter.
  • 19th Century : With the growth of schools and universities, assignments became more widespread and were used to assess student progress and achievement.
  • 20th Century: The rise of distance education and online learning led to the further development of assignments as an integral part of the educational process.
  • Present Day: Assignments continue to be used in a variety of educational settings and are seen as an effective way to promote student learning and assess student achievement. The nature and format of assignments continue to evolve in response to changing educational needs and technological innovations.

Types of Assignment

Here are some of the most common types of assignments:

An essay is a piece of writing that presents an argument, analysis, or interpretation of a topic or question. It usually consists of an introduction, body paragraphs, and a conclusion.

Essay structure:

  • Introduction : introduces the topic and thesis statement
  • Body paragraphs : each paragraph presents a different argument or idea, with evidence and analysis to support it
  • Conclusion : summarizes the key points and reiterates the thesis statement

Research paper

A research paper involves gathering and analyzing information on a particular topic, and presenting the findings in a well-structured, documented paper. It usually involves conducting original research, collecting data, and presenting it in a clear, organized manner.

Research paper structure:

  • Title page : includes the title of the paper, author’s name, date, and institution
  • Abstract : summarizes the paper’s main points and conclusions
  • Introduction : provides background information on the topic and research question
  • Literature review: summarizes previous research on the topic
  • Methodology : explains how the research was conducted
  • Results : presents the findings of the research
  • Discussion : interprets the results and draws conclusions
  • Conclusion : summarizes the key findings and implications

A case study involves analyzing a real-life situation, problem or issue, and presenting a solution or recommendations based on the analysis. It often involves extensive research, data analysis, and critical thinking.

Case study structure:

  • Introduction : introduces the case study and its purpose
  • Background : provides context and background information on the case
  • Analysis : examines the key issues and problems in the case
  • Solution/recommendations: proposes solutions or recommendations based on the analysis
  • Conclusion: Summarize the key points and implications

A lab report is a scientific document that summarizes the results of a laboratory experiment or research project. It typically includes an introduction, methodology, results, discussion, and conclusion.

Lab report structure:

  • Title page : includes the title of the experiment, author’s name, date, and institution
  • Abstract : summarizes the purpose, methodology, and results of the experiment
  • Methods : explains how the experiment was conducted
  • Results : presents the findings of the experiment

Presentation

A presentation involves delivering information, data or findings to an audience, often with the use of visual aids such as slides, charts, or diagrams. It requires clear communication skills, good organization, and effective use of technology.

Presentation structure:

  • Introduction : introduces the topic and purpose of the presentation
  • Body : presents the main points, findings, or data, with the help of visual aids
  • Conclusion : summarizes the key points and provides a closing statement

Creative Project

A creative project is an assignment that requires students to produce something original, such as a painting, sculpture, video, or creative writing piece. It allows students to demonstrate their creativity and artistic skills.

Creative project structure:

  • Introduction : introduces the project and its purpose
  • Body : presents the creative work, with explanations or descriptions as needed
  • Conclusion : summarizes the key elements and reflects on the creative process.

Examples of Assignments

Following are Examples of Assignment templates samples:

Essay template:

I. Introduction

  • Hook: Grab the reader’s attention with a catchy opening sentence.
  • Background: Provide some context or background information on the topic.
  • Thesis statement: State the main argument or point of your essay.

II. Body paragraphs

  • Topic sentence: Introduce the main idea or argument of the paragraph.
  • Evidence: Provide evidence or examples to support your point.
  • Analysis: Explain how the evidence supports your argument.
  • Transition: Use a transition sentence to lead into the next paragraph.

III. Conclusion

  • Restate thesis: Summarize your main argument or point.
  • Review key points: Summarize the main points you made in your essay.
  • Concluding thoughts: End with a final thought or call to action.

Research paper template:

I. Title page

  • Title: Give your paper a descriptive title.
  • Author: Include your name and institutional affiliation.
  • Date: Provide the date the paper was submitted.

II. Abstract

  • Background: Summarize the background and purpose of your research.
  • Methodology: Describe the methods you used to conduct your research.
  • Results: Summarize the main findings of your research.
  • Conclusion: Provide a brief summary of the implications and conclusions of your research.

III. Introduction

  • Background: Provide some background information on the topic.
  • Research question: State your research question or hypothesis.
  • Purpose: Explain the purpose of your research.

IV. Literature review

  • Background: Summarize previous research on the topic.
  • Gaps in research: Identify gaps or areas that need further research.

V. Methodology

  • Participants: Describe the participants in your study.
  • Procedure: Explain the procedure you used to conduct your research.
  • Measures: Describe the measures you used to collect data.

VI. Results

  • Quantitative results: Summarize the quantitative data you collected.
  • Qualitative results: Summarize the qualitative data you collected.

VII. Discussion

  • Interpretation: Interpret the results and explain what they mean.
  • Implications: Discuss the implications of your research.
  • Limitations: Identify any limitations or weaknesses of your research.

VIII. Conclusion

  • Review key points: Summarize the main points you made in your paper.

Case study template:

  • Background: Provide background information on the case.
  • Research question: State the research question or problem you are examining.
  • Purpose: Explain the purpose of the case study.

II. Analysis

  • Problem: Identify the main problem or issue in the case.
  • Factors: Describe the factors that contributed to the problem.
  • Alternative solutions: Describe potential solutions to the problem.

III. Solution/recommendations

  • Proposed solution: Describe the solution you are proposing.
  • Rationale: Explain why this solution is the best one.
  • Implementation: Describe how the solution can be implemented.

IV. Conclusion

  • Summary: Summarize the main points of your case study.

Lab report template:

  • Title: Give your report a descriptive title.
  • Date: Provide the date the report was submitted.
  • Background: Summarize the background and purpose of the experiment.
  • Methodology: Describe the methods you used to conduct the experiment.
  • Results: Summarize the main findings of the experiment.
  • Conclusion: Provide a brief summary of the implications and conclusions
  • Background: Provide some background information on the experiment.
  • Hypothesis: State your hypothesis or research question.
  • Purpose: Explain the purpose of the experiment.

IV. Materials and methods

  • Materials: List the materials and equipment used in the experiment.
  • Procedure: Describe the procedure you followed to conduct the experiment.
  • Data: Present the data you collected in tables or graphs.
  • Analysis: Analyze the data and describe the patterns or trends you observed.

VI. Discussion

  • Implications: Discuss the implications of your findings.
  • Limitations: Identify any limitations or weaknesses of the experiment.

VII. Conclusion

  • Restate hypothesis: Summarize your hypothesis or research question.
  • Review key points: Summarize the main points you made in your report.

Presentation template:

  • Attention grabber: Grab the audience’s attention with a catchy opening.
  • Purpose: Explain the purpose of your presentation.
  • Overview: Provide an overview of what you will cover in your presentation.

II. Main points

  • Main point 1: Present the first main point of your presentation.
  • Supporting details: Provide supporting details or evidence to support your point.
  • Main point 2: Present the second main point of your presentation.
  • Main point 3: Present the third main point of your presentation.
  • Summary: Summarize the main points of your presentation.
  • Call to action: End with a final thought or call to action.

Creative writing template:

  • Setting: Describe the setting of your story.
  • Characters: Introduce the main characters of your story.
  • Rising action: Introduce the conflict or problem in your story.
  • Climax: Present the most intense moment of the story.
  • Falling action: Resolve the conflict or problem in your story.
  • Resolution: Describe how the conflict or problem was resolved.
  • Final thoughts: End with a final thought or reflection on the story.

How to Write Assignment

Here is a general guide on how to write an assignment:

  • Understand the assignment prompt: Before you begin writing, make sure you understand what the assignment requires. Read the prompt carefully and make note of any specific requirements or guidelines.
  • Research and gather information: Depending on the type of assignment, you may need to do research to gather information to support your argument or points. Use credible sources such as academic journals, books, and reputable websites.
  • Organize your ideas : Once you have gathered all the necessary information, organize your ideas into a clear and logical structure. Consider creating an outline or diagram to help you visualize your ideas.
  • Write a draft: Begin writing your assignment using your organized ideas and research. Don’t worry too much about grammar or sentence structure at this point; the goal is to get your thoughts down on paper.
  • Revise and edit: After you have written a draft, revise and edit your work. Make sure your ideas are presented in a clear and concise manner, and that your sentences and paragraphs flow smoothly.
  • Proofread: Finally, proofread your work for spelling, grammar, and punctuation errors. It’s a good idea to have someone else read over your assignment as well to catch any mistakes you may have missed.
  • Submit your assignment : Once you are satisfied with your work, submit your assignment according to the instructions provided by your instructor or professor.

Applications of Assignment

Assignments have many applications across different fields and industries. Here are a few examples:

  • Education : Assignments are a common tool used in education to help students learn and demonstrate their knowledge. They can be used to assess a student’s understanding of a particular topic, to develop critical thinking skills, and to improve writing and research abilities.
  • Business : Assignments can be used in the business world to assess employee skills, to evaluate job performance, and to provide training opportunities. They can also be used to develop business plans, marketing strategies, and financial projections.
  • Journalism : Assignments are often used in journalism to produce news articles, features, and investigative reports. Journalists may be assigned to cover a particular event or topic, or to research and write a story on a specific subject.
  • Research : Assignments can be used in research to collect and analyze data, to conduct experiments, and to present findings in written or oral form. Researchers may be assigned to conduct research on a specific topic, to write a research paper, or to present their findings at a conference or seminar.
  • Government : Assignments can be used in government to develop policy proposals, to conduct research, and to analyze data. Government officials may be assigned to work on a specific project or to conduct research on a particular topic.
  • Non-profit organizations: Assignments can be used in non-profit organizations to develop fundraising strategies, to plan events, and to conduct research. Volunteers may be assigned to work on a specific project or to help with a particular task.

Purpose of Assignment

The purpose of an assignment varies depending on the context in which it is given. However, some common purposes of assignments include:

  • Assessing learning: Assignments are often used to assess a student’s understanding of a particular topic or concept. This allows educators to determine if a student has mastered the material or if they need additional support.
  • Developing skills: Assignments can be used to develop a wide range of skills, such as critical thinking, problem-solving, research, and communication. Assignments that require students to analyze and synthesize information can help to build these skills.
  • Encouraging creativity: Assignments can be designed to encourage students to be creative and think outside the box. This can help to foster innovation and original thinking.
  • Providing feedback : Assignments provide an opportunity for teachers to provide feedback to students on their progress and performance. Feedback can help students to understand where they need to improve and to develop a growth mindset.
  • Meeting learning objectives : Assignments can be designed to help students meet specific learning objectives or outcomes. For example, a writing assignment may be designed to help students improve their writing skills, while a research assignment may be designed to help students develop their research skills.

When to write Assignment

Assignments are typically given by instructors or professors as part of a course or academic program. The timing of when to write an assignment will depend on the specific requirements of the course or program, but in general, assignments should be completed within the timeframe specified by the instructor or program guidelines.

It is important to begin working on assignments as soon as possible to ensure enough time for research, writing, and revisions. Waiting until the last minute can result in rushed work and lower quality output.

It is also important to prioritize assignments based on their due dates and the amount of work required. This will help to manage time effectively and ensure that all assignments are completed on time.

In addition to assignments given by instructors or professors, there may be other situations where writing an assignment is necessary. For example, in the workplace, assignments may be given to complete a specific project or task. In these situations, it is important to establish clear deadlines and expectations to ensure that the assignment is completed on time and to a high standard.

Characteristics of Assignment

Here are some common characteristics of assignments:

  • Purpose : Assignments have a specific purpose, such as assessing knowledge or developing skills. They are designed to help students learn and achieve specific learning objectives.
  • Requirements: Assignments have specific requirements that must be met, such as a word count, format, or specific content. These requirements are usually provided by the instructor or professor.
  • Deadline: Assignments have a specific deadline for completion, which is usually set by the instructor or professor. It is important to meet the deadline to avoid penalties or lower grades.
  • Individual or group work: Assignments can be completed individually or as part of a group. Group assignments may require collaboration and communication with other group members.
  • Feedback : Assignments provide an opportunity for feedback from the instructor or professor. This feedback can help students to identify areas of improvement and to develop their skills.
  • Academic integrity: Assignments require academic integrity, which means that students must submit original work and avoid plagiarism. This includes citing sources properly and following ethical guidelines.
  • Learning outcomes : Assignments are designed to help students achieve specific learning outcomes. These outcomes are usually related to the course objectives and may include developing critical thinking skills, writing abilities, or subject-specific knowledge.

Advantages of Assignment

There are several advantages of assignment, including:

  • Helps in learning: Assignments help students to reinforce their learning and understanding of a particular topic. By completing assignments, students get to apply the concepts learned in class, which helps them to better understand and retain the information.
  • Develops critical thinking skills: Assignments often require students to think critically and analyze information in order to come up with a solution or answer. This helps to develop their critical thinking skills, which are important for success in many areas of life.
  • Encourages creativity: Assignments that require students to create something, such as a piece of writing or a project, can encourage creativity and innovation. This can help students to develop new ideas and perspectives, which can be beneficial in many areas of life.
  • Builds time-management skills: Assignments often come with deadlines, which can help students to develop time-management skills. Learning how to manage time effectively is an important skill that can help students to succeed in many areas of life.
  • Provides feedback: Assignments provide an opportunity for students to receive feedback on their work. This feedback can help students to identify areas where they need to improve and can help them to grow and develop.

Limitations of Assignment

There are also some limitations of assignments that should be considered, including:

  • Limited scope: Assignments are often limited in scope, and may not provide a comprehensive understanding of a particular topic. They may only cover a specific aspect of a topic, and may not provide a full picture of the subject matter.
  • Lack of engagement: Some assignments may not engage students in the learning process, particularly if they are repetitive or not challenging enough. This can lead to a lack of motivation and interest in the subject matter.
  • Time-consuming: Assignments can be time-consuming, particularly if they require a lot of research or writing. This can be a disadvantage for students who have other commitments, such as work or extracurricular activities.
  • Unreliable assessment: The assessment of assignments can be subjective and may not always accurately reflect a student’s understanding or abilities. The grading may be influenced by factors such as the instructor’s personal biases or the student’s writing style.
  • Lack of feedback : Although assignments can provide feedback, this feedback may not always be detailed or useful. Instructors may not have the time or resources to provide detailed feedback on every assignment, which can limit the value of the feedback that students receive.

About the author

' src=

Muhammad Hassan

Researcher, Academic Writer, Web developer

You may also like

Research Objectives

Research Objectives – Types, Examples and...

Future Research

Future Research – Thesis Guide

Background of The Study

Background of The Study – Examples and Writing...

Data Verification

Data Verification – Process, Types and Examples

Limitations in Research

Limitations in Research – Types, Examples and...

Research Questions

Research Questions – Types, Examples and Writing...

MBA Notes

How to Solve the Assignment Problem: A Complete Guide

Table of Contents

Assignment problem is a special type of linear programming problem that deals with assigning a number of resources to an equal number of tasks in the most efficient way. The goal is to minimize the total cost of assignments while ensuring that each task is assigned to only one resource and each resource is assigned to only one task. In this blog, we will discuss the solution of the assignment problem using the Hungarian method, which is a popular algorithm for solving the problem.

Understanding the Assignment Problem

Before we dive into the solution, it is important to understand the problem itself. In the assignment problem, we have a matrix of costs, where each row represents a resource and each column represents a task. The objective is to assign each resource to a task in such a way that the total cost of assignments is minimized. However, there are certain constraints that need to be satisfied – each resource can be assigned to only one task and each task can be assigned to only one resource.

Solving the Assignment Problem

There are various methods for solving the assignment problem, including the Hungarian method, the brute force method, and the auction algorithm. Here, we will focus on the steps involved in solving the assignment problem using the Hungarian method, which is the most commonly used and efficient method.

Step 1: Set up the cost matrix

The first step in solving the assignment problem is to set up the cost matrix, which represents the cost of assigning a task to an agent. The matrix should be square and have the same number of rows and columns as the number of tasks and agents, respectively.

Step 2: Subtract the smallest element from each row and column

To simplify the calculations, we need to reduce the size of the cost matrix by subtracting the smallest element from each row and column. This step is called matrix reduction.

Step 3: Cover all zeros with the minimum number of lines

The next step is to cover all zeros in the matrix with the minimum number of horizontal and vertical lines. This step is called matrix covering.

Step 4: Test for optimality and adjust the matrix

To test for optimality, we need to calculate the minimum number of lines required to cover all zeros in the matrix. If the number of lines equals the number of rows or columns, the solution is optimal. If not, we need to adjust the matrix and repeat steps 3 and 4 until we get an optimal solution.

Step 5: Assign the tasks to the agents

The final step is to assign the tasks to the agents based on the optimal solution obtained in step 4. This will give us the most cost-effective or profit-maximizing assignment.

Solution of the Assignment Problem using the Hungarian Method

The Hungarian method is an algorithm that uses a step-by-step approach to find the optimal assignment. The algorithm consists of the following steps:

  • Subtract the smallest entry in each row from all the entries of the row.
  • Subtract the smallest entry in each column from all the entries of the column.
  • Draw the minimum number of lines to cover all zeros in the matrix. If the number of lines drawn is equal to the number of rows, we have an optimal solution. If not, go to step 4.
  • Determine the smallest entry not covered by any line. Subtract it from all uncovered entries and add it to all entries covered by two lines. Go to step 3.

The above steps are repeated until an optimal solution is obtained. The optimal solution will have all zeros covered by the minimum number of lines. The assignments can be made by selecting the rows and columns with a single zero in the final matrix.

Applications of the Assignment Problem

The assignment problem has various applications in different fields, including computer science, economics, logistics, and management. In this section, we will provide some examples of how the assignment problem is used in real-life situations.

Applications in Computer Science

The assignment problem can be used in computer science to allocate resources to different tasks, such as allocating memory to processes or assigning threads to processors.

Applications in Economics

The assignment problem can be used in economics to allocate resources to different agents, such as allocating workers to jobs or assigning projects to contractors.

Applications in Logistics

The assignment problem can be used in logistics to allocate resources to different activities, such as allocating vehicles to routes or assigning warehouses to customers.

Applications in Management

The assignment problem can be used in management to allocate resources to different projects, such as allocating employees to tasks or assigning budgets to departments.

Let’s consider the following scenario: a manager needs to assign three employees to three different tasks. Each employee has different skills, and each task requires specific skills. The manager wants to minimize the total time it takes to complete all the tasks. The skills and the time required for each task are given in the table below:

Task 1 Task 2 Task 3
Emp 1 5 7 6
Emp 2 6 4 5
Emp 3 8 5 3

The assignment problem is to determine which employee should be assigned to which task to minimize the total time required. To solve this problem, we can use the Hungarian method, which we discussed in the previous blog.

Using the Hungarian method, we first subtract the smallest entry in each row from all the entries of the row:

Task 1 Task 2 Task 3
Emp 1 0 2 1
Emp 2 2 0 1
Emp 3 5 2 0

Next, we subtract the smallest entry in each column from all the entries of the column:

Task 1 Task 2 Task 3
Emp 1 0 2 1
Emp 2 2 0 1
Emp 3 5 2 0
0 0 0

We draw the minimum number of lines to cover all the zeros in the matrix, which in this case is three:

Since the number of lines is equal to the number of rows, we have an optimal solution. The assignments can be made by selecting the rows and columns with a single zero in the final matrix. In this case, the optimal assignments are:

  • Emp 1 to Task 3
  • Emp 2 to Task 2
  • Emp 3 to Task 1

This assignment results in a total time of 9 units.

I hope this example helps you better understand the assignment problem and how to solve it using the Hungarian method.

Solving the assignment problem may seem daunting, but with the right approach, it can be a straightforward process. By following the steps outlined in this guide, you can confidently tackle any assignment problem that comes your way.

How useful was this post?

Click on a star to rate it!

Average rating 0 / 5. Vote count: 0

No votes so far! Be the first to rate this post.

We are sorry that this post was not useful for you! 😔

Let us improve this post!

Tell us how we can improve this post?

Operations Research

1 Operations Research-An Overview

  • History of O.R.
  • Approach, Techniques and Tools
  • Phases and Processes of O.R. Study
  • Typical Applications of O.R
  • Limitations of Operations Research
  • Models in Operations Research
  • O.R. in real world

2 Linear Programming: Formulation and Graphical Method

  • General formulation of Linear Programming Problem
  • Optimisation Models
  • Basics of Graphic Method
  • Important steps to draw graph
  • Multiple, Unbounded Solution and Infeasible Problems
  • Solving Linear Programming Graphically Using Computer
  • Application of Linear Programming in Business and Industry

3 Linear Programming-Simplex Method

  • Principle of Simplex Method
  • Computational aspect of Simplex Method
  • Simplex Method with several Decision Variables
  • Two Phase and M-method
  • Multiple Solution, Unbounded Solution and Infeasible Problem
  • Sensitivity Analysis
  • Dual Linear Programming Problem

4 Transportation Problem

  • Basic Feasible Solution of a Transportation Problem
  • Modified Distribution Method
  • Stepping Stone Method
  • Unbalanced Transportation Problem
  • Degenerate Transportation Problem
  • Transhipment Problem
  • Maximisation in a Transportation Problem

5 Assignment Problem

  • Solution of the Assignment Problem
  • Unbalanced Assignment Problem
  • Problem with some Infeasible Assignments
  • Maximisation in an Assignment Problem
  • Crew Assignment Problem

6 Application of Excel Solver to Solve LPP

  • Building Excel model for solving LP: An Illustrative Example

7 Goal Programming

  • Concepts of goal programming
  • Goal programming model formulation
  • Graphical method of goal programming
  • The simplex method of goal programming
  • Using Excel Solver to Solve Goal Programming Models
  • Application areas of goal programming

8 Integer Programming

  • Some Integer Programming Formulation Techniques
  • Binary Representation of General Integer Variables
  • Unimodularity
  • Cutting Plane Method
  • Branch and Bound Method
  • Solver Solution

9 Dynamic Programming

  • Dynamic Programming Methodology: An Example
  • Definitions and Notations
  • Dynamic Programming Applications

10 Non-Linear Programming

  • Solution of a Non-linear Programming Problem
  • Convex and Concave Functions
  • Kuhn-Tucker Conditions for Constrained Optimisation
  • Quadratic Programming
  • Separable Programming
  • NLP Models with Solver

11 Introduction to game theory and its Applications

  • Important terms in Game Theory
  • Saddle points
  • Mixed strategies: Games without saddle points
  • 2 x n games
  • Exploiting an opponent’s mistakes

12 Monte Carlo Simulation

  • Reasons for using simulation
  • Monte Carlo simulation
  • Limitations of simulation
  • Steps in the simulation process
  • Some practical applications of simulation
  • Two typical examples of hand-computed simulation
  • Computer simulation

13 Queueing Models

  • Characteristics of a queueing model
  • Notations and Symbols
  • Statistical methods in queueing
  • The M/M/I System
  • The M/M/C System
  • The M/Ek/I System
  • Decision problems in queueing

AccountingTools

Accounting CPE Courses & Books

Cost assignment definition

What is cost assignment.

Cost assignment is the allocation of costs to the activities or objects that triggered the incurrence of the costs. The concept is heavily used in activity-based costing , where overhead costs are traced back to the actions causing the overhead to be incurred. The cost assignment is based on one or more cost drivers .

Cost assignments are associated with direct costs and indirect costs. Assignments of costs differ, based on which cost has been incurred. These differences are as follows:

Assignment of direct costs . Direct costs can be traced directly to a cost object. For example, the valve used on a basketball is a direct cost of the basketball, since it is an item that is directly incorporated into the product. These costs are assigned to cost objects based on a bill of materials (in the case of materials) or a time sheet (in the case of labor).

Assignment of indirect costs . Indirect costs cannot be traced directly to a cost object. For example, the cost of the materials manager who schedules production activities for a number of different products cannot be directly assigned to each of the products manufactured. Instead, these costs are assigned to cost objects using a reasonable basis of allocation.

Example of a Cost Assignment

A university operates its own maintenance department; the cost of the department is assigned to the various other departments of the university based on their consumption of the department's maintenance services.

Terms Similar to Cost Assignment

Cost assignment is also known as cost allocation .

Related AccountingTools Courses

Activity-Based Costing

Cost Accounting Fundamentals

Related Articles

Accounting Allocation the Easy Way (podcast)

Activity Cost Assignment

Accounting Books

College Textbooks

Finance Books

Operations Books

Copyright 2024

Your Article Library

Assignment problem in linear programming : introduction and assignment model.

what is the assignment methods

ADVERTISEMENTS:

Assignment problem is a special type of linear programming problem which deals with the allocation of the various resources to the various activities on one to one basis. It does it in such a way that the cost or time involved in the process is minimum and profit or sale is maximum. Though there problems can be solved by simplex method or by transportation method but assignment model gives a simpler approach for these problems.

In a factory, a supervisor may have six workers available and six jobs to fire. He will have to take decision regarding which job should be given to which worker. Problem forms one to one basis. This is an assignment problem.

1. Assignment Model :

Suppose there are n facilitates and n jobs it is clear that in this case, there will be n assignments. Each facility or say worker can perform each job, one at a time. But there should be certain procedure by which assignment should be made so that the profit is maximized or the cost or time is minimized.

job of Work

In the table, Co ij is defined as the cost when j th job is assigned to i th worker. It maybe noted here that this is a special case of transportation problem when the number of rows is equal to number of columns.

Mathematical Formulation:

Any basic feasible solution of an Assignment problem consists (2n – 1) variables of which the (n – 1) variables are zero, n is number of jobs or number of facilities. Due to this high degeneracy, if we solve the problem by usual transportation method, it will be a complex and time consuming work. Thus a separate technique is derived for it. Before going to the absolute method it is very important to formulate the problem.

Suppose x jj is a variable which is defined as

1 if the i th job is assigned to j th machine or facility

0 if the i th job is not assigned to j th machine or facility.

Now as the problem forms one to one basis or one job is to be assigned to one facility or machine.

Assignment Model

The total assignment cost will be given by

clip_image005

The above definition can be developed into mathematical model as follows:

Determine x ij > 0 (i, j = 1,2, 3…n) in order to

Assignment Model

Subjected to constraints

Assignment Model

and x ij is either zero or one.

Method to solve Problem (Hungarian Technique):

Consider the objective function of minimization type. Following steps are involved in solving this Assignment problem,

1. Locate the smallest cost element in each row of the given cost table starting with the first row. Now, this smallest element is subtracted form each element of that row. So, we will be getting at least one zero in each row of this new table.

2. Having constructed the table (as by step-1) take the columns of the table. Starting from first column locate the smallest cost element in each column. Now subtract this smallest element from each element of that column. Having performed the step 1 and step 2, we will be getting at least one zero in each column in the reduced cost table.

3. Now, the assignments are made for the reduced table in following manner.

(i) Rows are examined successively, until the row with exactly single (one) zero is found. Assignment is made to this single zero by putting square □ around it and in the corresponding column, all other zeros are crossed out (x) because these will not be used to make any other assignment in this column. Step is conducted for each row.

(ii) Step 3 (i) in now performed on the columns as follow:- columns are examined successively till a column with exactly one zero is found. Now , assignment is made to this single zero by putting the square around it and at the same time, all other zeros in the corresponding rows are crossed out (x) step is conducted for each column.

(iii) Step 3, (i) and 3 (ii) are repeated till all the zeros are either marked or crossed out. Now, if the number of marked zeros or the assignments made are equal to number of rows or columns, optimum solution has been achieved. There will be exactly single assignment in each or columns without any assignment. In this case, we will go to step 4.

4. At this stage, draw the minimum number of lines (horizontal and vertical) necessary to cover all zeros in the matrix obtained in step 3, Following procedure is adopted:

(iii) Now tick mark all the rows that are not already marked and that have assignment in the marked columns.

(iv) All the steps i.e. (4(i), 4(ii), 4(iii) are repeated until no more rows or columns can be marked.

(v) Now draw straight lines which pass through all the un marked rows and marked columns. It can also be noticed that in an n x n matrix, always less than ‘n’ lines will cover all the zeros if there is no solution among them.

5. In step 4, if the number of lines drawn are equal to n or the number of rows, then it is the optimum solution if not, then go to step 6.

6. Select the smallest element among all the uncovered elements. Now, this element is subtracted from all the uncovered elements and added to the element which lies at the intersection of two lines. This is the matrix for fresh assignments.

7. Repeat the procedure from step (3) until the number of assignments becomes equal to the number of rows or number of columns.

Related Articles:

  • Two Phase Methods of Problem Solving in Linear Programming: First and Second Phase
  • Linear Programming: Applications, Definitions and Problems

No comments yet.

Leave a reply click here to cancel reply..

You must be logged in to post a comment.

web statistics

Quantitative Techniques: Theory and Problems by P. C. Tulsian, Vishal Pandey

Get full access to Quantitative Techniques: Theory and Problems and 60K+ other titles, with a free 10-day trial of O'Reilly.

There are also live events, courses curated by job role, and more.

WHAT IS ASSIGNMENT PROBLEM

Assignment Problem is a special type of linear programming problem where the objective is to minimise the cost or time of completing a number of jobs by a number of persons.

The assignment problem in the general form can be stated as follows:

“Given n facilities, n jobs and the effectiveness of each facility for each job, the problem is to assign each facility to one and only one job in such a way that the measure of effectiveness is optimised (Maximised or Minimised).”

Several problems of management has a structure identical with the assignment problem.

Example I A manager has four persons (i.e. facilities) available for four separate jobs (i.e. jobs) and the cost of assigning (i.e. effectiveness) each job to each ...

Get Quantitative Techniques: Theory and Problems now with the O’Reilly learning platform.

O’Reilly members experience books, live events, courses curated by job role, and more from O’Reilly and nearly 200 top publishers.

Don’t leave empty-handed

Get Mark Richards’s Software Architecture Patterns ebook to better understand how to design components—and how they should interact.

It’s yours, free.

Cover of Software Architecture Patterns

Check it out now on O’Reilly

Dive in for free with a 10-day trial of the O’Reilly learning platform—then explore all the other resources our members count on to build skills and solve problems every day.

what is the assignment methods

MBA Knowledge Base

Business • Management • Technology

Home » Management Science » Transportation and Assignment Models in Operations Research

Transportation and Assignment Models in Operations Research

Transportation and assignment models are special purpose algorithms of the linear programming. The simplex method of Linear Programming Problems(LPP) proves to be inefficient is certain situations like determining optimum assignment of jobs to persons, supply of materials from several supply points to several destinations and the like. More effective solution models have been evolved and these are called assignment and transportation models.

The transportation model is concerned with selecting the routes between supply and demand points in order to minimize costs of transportation subject to constraints of supply at any supply point and demand at any demand point. Assume a company has 4 manufacturing plants with different capacity levels, and 5 regional distribution centres. 4 x 5 = 20 routes are possible. Given the transportation costs per load of each of 20 routes between the manufacturing (supply) plants and the regional distribution (demand) centres, and supply and demand constraints, how many loads can be transported through different routes so as to minimize transportation costs? The answer to this question is obtained easily through the transportation algorithm.

Similarly, how are we to assign different jobs to different persons/machines, given cost of job completion for each pair of job machine/person? The objective is minimizing total cost. This is best solved through assignment algorithm.

Uses of Transportation and Assignment Models in Decision Making

The broad purposes of Transportation and Assignment models in LPP are just mentioned above. Now we have just enumerated the different situations where we can make use of these models.

Transportation model is used in the following:

  • To decide the transportation of new materials from various centres to different manufacturing plants. In the case of multi-plant company this is highly useful.
  • To decide the transportation of finished goods from different manufacturing plants to the different distribution centres. For a multi-plant-multi-market company this is useful.
  • To decide the transportation of finished goods from different manufacturing plants to the different distribution centres. For a multi-plant-multi-market company this is useful. These two are the uses of transportation model. The objective is minimizing transportation cost.

Assignment model is used in the following:

  • To decide the assignment of jobs to persons/machines, the assignment model is used.
  • To decide the route a traveling executive has to adopt (dealing with the order inn which he/she has to visit different places).
  • To decide the order in which different activities performed on one and the same facility be taken up.

In the case of transportation model, the supply quantity may be less or more than the demand. Similarly the assignment model, the number of jobs may be equal to, less or more than the number of machines/persons available. In all these cases the simplex method of LPP can be adopted, but transportation and assignment models are more effective, less time consuming and easier than the LPP.

Related posts:

  • Operations Research approach of problem solving
  • Introduction to Transportation Problem
  • Procedure for finding an optimum solution for transportation problem
  • Initial basic feasible solution of a transportation problem
  • Top 7 Best Ways of Getting MBA Assignment Writing Help
  • Introduction to Decision Models
  • Transportation Cost Elements
  • Modes of Transportation in Logistics
  • Factors Affecting Transportation in Logistics
  • Export/Import Transportation Systems

One thought on “ Transportation and Assignment Models in Operations Research ”

Exclussive dff. And easy understude

Leave a Reply Cancel reply

Your email address will not be published. Required fields are marked *

For each employee assignment, you can enter one or more payment methods, selecting from the list of valid methods for the employee's payroll.

If you enter multiple methods (for example because the employee is paid from more than one source account), you can enter the proportion of pay for each method and its processing priority. An assignment can have two instances of the same payment method, for example if salary is divided between two bank accounts.

Employees with no personal payment method on record receive pay by the default payment method of their payrolls.

You enter payment methods for employee assignments in the Personal Payment Method window. You can also use this window to enter the payee for third party payments.

To enter payment methods you must:

Assign the employee to a payroll.

For third party payment methods, define the payee in the Organization window (using the classification Payee Organization) or the Contact window (using the relationship Payments Recipient).

Set your effective date to the date on which to begin paying the employee by this method.

In the Name field, select a payment method.

Enter a number in the Priority field to determine the order for Oracle Payroll to use each payment method. It uses the method with the lowest number first.

When you select the payment type, the application automatically displays the balance currency and payment currency fields depending on the currency defined at the business group level.

Enter either the amount or percentage of the assignment's pay to be paid by this method.

If the total of the amounts you allocate to payment methods is less than the amount to be paid, Oracle Payroll uses the payment method with the highest priority number to pay the excess.

For UAE users only: To specify the method of payment for an employee's end of service remuneration or pension, enter the payment method details, click the Further Information field, and then select Yes.

For everywhere apart from the UK, if the payment method is a magnetic transfer type, open the Bank Details window for entry of information about the employee's bank account.

Set your effective date to the date on which to begin making payments using this method.

In the Name field, select a third party payment method.

Third party payment methods automatically receive priority 1 (the highest priority) and you cannot change this. You cannot split a third party payment between different payment methods; the Percentage field always displays 100%.

In the Payee region, select an organization or a person.

Save your work.

You can select this third party payment method in the Payee Details entry value when you enter a deduction to be paid as a third party payment.

One bed, two blankets: We put the Scandinavian sleep method to the test

If you sleep with a cover hog, this might be for you.

what is the assignment methods

There are plenty of Scandinavian exports worth embracing, including saunas , Lego , Dansk tableware , the Billy bookcase , Kransekake , Marimekko . So when the Scandinavian sleep method started popping up everywhere on social media , I took note.

The idea: Two people share a bed but use separate covers. The promise: a better night’s sleep. The bedding arrangement is popular in other European countries as well; on a trip to Paris a couple of years ago, our hotel room bed was outfitted with two sets of blankets and top sheets.

But does the method work? And how do you make a bed with two blankets? We put it to the test. We gave five couples two twin-size down alternative comforters each and asked them to try the method for a week. Here’s what they had to say.

Typical sleeping arrangement: Queen-size bed with a flat sheet, blanket and comforter.

Pain points: Tester 1’s husband accuses her of being a chronic sheet stealer, but she thinks the main issue is that they operate at different temperatures: “I am a nuclear furnace when I sleep, while my husband is a block of ice,” she says. Regardless, it seems like someone is always getting shortchanged on covers, or someone is tossing and turning.

How they carried out the test: They used the separate comforters, no flat sheet or blankets.

The experience: She likes to throw her foot or leg out from under the covers when she gets too hot, and the separate covers allowed her to do this on both sides, instead of just one. Another bonus: Lounging in bed on weekends was better with the extra covers. “It was absolutely luxuriant to take over the whole bed and both comforters and do the crossword puzzle,” she says. Her husband liked that he could roll over and move around in bed without affecting his wife.

The bed-making problem: They don’t typically make their bed, so this wasn’t an issue. “No shade on anyone who is put together enough in the morning to arrange their bedding, but we usually leave it in a chaotic state, and tend to reassemble the tangle of sheets into something more coherent as we’re going to bed, not when we wake up,” she says.

The verdict: The arrangement worked well for them, and they may implement it in the future. “The only hesitation is that we have so much bedding, buying more almost becomes a storage problem at this point,” she says. “But I think we slept better and more comfortably, so if I see twin comforters on sale anytime soon, there’s a good chance we will make the switch.”

Typical sleeping arrangement: Queen-size bed with a top sheet and a duvet.

Pain points: Tester 2 and her husband often go to bed at different times, and many nights one of them has to get up to tend to their toddler, which can be disruptive. There are also issues with sharing covers, she says: “One of us (a.k.a. me) is allegedly a cover hog, which means the other of us sleeps in a defensive crouch with respect to the shared covers as a way to maintain some blanket.”

How they carried out the test: They started with the two twin-size down alternative duvets provided, but her husband didn’t like the texture, so he swapped his out for a twin-size quilt they had. And she realized she preferred a bigger blanket, so she swapped in a queen-size quilt. They kept their queen-size flat sheet.

The experience: The test went well for this couple. Even the difference of opinions on the blankets ended up working in their favor. “The idea that we could each seek out a blanket with our preferred texture and warmth level was an improvement on sharing one quilt,” she says. “It was great to each have our own quilt and made for more restful sleeping.”

The bed-making problem: They don’t make the bed daily but like to do it when time allows. Settling on one twin quilt for him and a queen-size quilt for her helped here. During the day, they made the bed with the queen-size cover and folded the twin cover across the bottom of the bed.

The verdict: They said the arrangement was a big improvement on their previous blanket-sharing situation and plan to continue to use it.

Typical sleeping arrangement: Queen-size bed with a top sheet and comforter.

Pain points: Tester 3 says her husband is an occasional cover-stealer, and she’s a light sleeper who wakes easily if he is tossing and turning.

How they carried out the test: They used the twin comforters and skipped the flat sheet.

The experience: Her husband tossed and turned a good bit one night, and got up to use the bathroom several times on another night, but it didn’t disturb her. She says, though, that she’s so used to his restlessness that she might not have been disturbed even while sharing covers. She liked being able to stick her leg out from under the covers on either side. But there were also drawbacks, including getting overheated. “I think during the night, the excess parts of our blankets would overlap on one person or the other,” she says.

The bed-making problem: They folded the twin comforters lengthwise and placed them side by side to give the bed a neat appearance.

The verdict: They plan to keep their original bedding configuration. “I don’t think we saw enough of a positive impact to switch to two blankets,” she says. She also likes having a neatly made bed, which was another strike against the Scandinavian method.

Typical sleeping arrangement: King-size bed with a flat sheet, blanket and duvet.

Pain points: Tester 4 likes to have covers tucked snugly around her, while her husband struggles to keep his side tucked. Sometimes that leads to uneven distribution of blankets. She maintains that because his covers are untucked, they bunch up at the bottom, creating the impression that she’s stolen them when, in fact, she has not. He admits this possibility: “I think I kick covers off and then overcompensate when trying to recover (ha),” he says. “Then the groggy tug-of-war begins.”

How they carried out the test: They used the two twin duvets provided and dragged out old twin-size top sheets from when their kids were little, giving them completely separate covers.

The experience: The test took cover-stealing out of the equation, and she reports that she was less aware of when her husband got in and out of bed (he goes to bed later — and gets up earlier — than she does). “Something has to go pretty wrong for you to end up with the other person’s covers,” her husband says. “I don’t really see a downside to it. It’s the happy medium between the freedom of separate beds and the traditional way that just doesn’t work for many people.” It didn’t solve his issues with keeping his covers in order; sometimes he woke up with just the duvet and couldn’t find the top sheet. She liked the fact that he couldn’t blame her for this problem anymore. Overall, it improved their quality of sleep.

The bed-making problem: They typically don’t make their bed, so this wasn’t an issue.

The verdict: They have returned to their usual sleeping arrangements for now, but may switch at some point in the future if they find bedding they like.

Typical sleeping arrangement: A queen bed with a flat sheet, blanket and comforter.

Pain points: Tester 5 says she and her partner’s covers come untucked at times, but overall, they don’t have issues with sharing. She sleeps “warm,” so prefers a cool room with a fan pointing at her, and often wakes up under just the sheet.

How they carried out the test: They started with just the separate comforters but missed having a top sheet, so after a week, they brought back a shared queen-size sheet.

The experience: While they slept well during the test, they didn’t notice much of a difference between sharing a comforter or having separate ones. “We already have a comforter we like enough, and both of us prefer to have a flat sheet between the comforter and ourselves,” she says. After a few nights, they added a sheet and blanket for her partner, who sleeps cooler. And after a week, they brought back the queen-size top sheet. Overall, they didn’t notice much of a difference sleeping with two duvets, possibly because they don’t struggle with sharing covers.

The bed-making problem: It’s easier to make the bed with fewer covers, she says, but it definitely looked less tidy. When her parents came to visit, they tossed their regular comforter over the bed for a neater appearance.

The verdict: They felt pretty neutral about the test and have gone back to their regular sleeping arrangement. “I think this is fine when we travel … but isn’t a huge benefit at home,” she says.

what is the assignment methods

ACM Digital Library home

  • Advanced Search

Transformer-based assignment decision network for multiple object tracking

New citation alert added.

This alert has been successfully added and will be sent to:

You will be notified whenever a record that you have chosen has been cited.

To manage your alert preferences, click on the button below.

New Citation Alert!

Please log in to your account

Information & Contributors

Bibliometrics & citations, view options, recommendations, heterogeneous fusion of omnidirectional and ptz cameras for multiple object tracking.

Dual-camera systems have been widely used in surveillance because of the ability to explore the wide field of view (FOV) of the omnidirectional camera and the wide zoom range of the PTZ camera. Most existing algorithms require a priori knowledge of the ...

Non-myopic information theoretic sensor management of a single pan-tilt-zoom camera for multiple object detection and tracking

Detailed derivation of an information theoretic framework for real PTZ management.Introduction and implementation of a non-myopic strategy.Large experimental validation, with synthetic and realistic datasets.Working demonstration of myopic strategy on ...

Hi-ROS: Open-source multi-camera sensor fusion for real-time people tracking

This paper presents Hi-ROS (Human Interaction in ROS), an open source framework focused on real-time accurate assessment of human motion. The system offers a series of tools to track multiple people in real-time by exploiting a calibrated camera ...

  • Open-source multi-camera multi-people tracking framework.
  • Support different types of cameras and pose estimation algorithms.
  • No assumptions about the number of people or cameras.
  • Tracked body poses are provided in real-time.

Information

Published in.

Elsevier Science Inc.

United States

Publication History

Author tags.

  • Multiple object tracking
  • MOTChallenge
  • Research-article

Contributors

Other metrics, bibliometrics, article metrics.

  • 0 Total Citations
  • 0 Total Downloads
  • Downloads (Last 12 months) 0
  • Downloads (Last 6 weeks) 0

View options

Login options.

Check if you have access through your login credentials or your institution to get full access on this article.

Full Access

Share this publication link.

Copying failed.

Share on social media

Affiliations, export citations.

  • Please download or close your previous search result export first before starting a new bulk export. Preview is not available. By clicking download, a status dialog will open to start the export process. The process may take a few minutes but once it finishes a file will be downloadable from your browser. You may continue to browse the DL while the export process is in progress. Download
  • Download citation
  • Copy citation

We are preparing your search results for download ...

We will inform you here when the file is ready.

Your file of search results citations is now ready.

Your search export query has expired. Please try again.

IMAGES

  1. Most Common Methodologies used in Assignment Writing

    what is the assignment methods

  2. Assignment Method

    what is the assignment methods

  3. Assignment Method

    what is the assignment methods

  4. What is an Assignment Method?

    what is the assignment methods

  5. PPT

    what is the assignment methods

  6. How to Write an Assignment: Step by Step Guide

    what is the assignment methods

VIDEO

  1. Chapter 9: Traffic Assignment Model

  2. Assignment 3

  3. Day 3(3)

  4. BSOC 134

  5. "Mastering Assignment Operators in Python: A Comprehensive Guide"

  6. CS636 Formal Methods Assignment 2 Spring 2024 Virtual University of Pakistan

COMMENTS

  1. Assignment Method

    Assignment method calculators, if reliable, can be used for the same. The simplex method can be solved as a linear programming problem using the simplex algorithm. The transportation method is a special case of the assignment problem. The method is, however, computationally inefficient for solving the assignment problem due to the solution's ...

  2. Assignment Method: Examples of How Resources Are Allocated

    Assignment Method: A method of allocating organizational resources. The assignment method is used to determine what resources are assigned to which department, machine or center of operation in ...

  3. Assignment method definition

    The assignment method is any technique used to assign organizational resources to activities. The best assignment method will maximize profits, typically through cost controls, increases in efficiency levels, and better management of bottleneck operations. The assignment method is incorporated into an organization's budgeting process, so that ...

  4. The Assignment Method: Definition, Applications, and ...

    The assignment method is a systematic framework used by organizations to allocate resources efficiently across different tasks, departments, or projects. Its effectiveness lies in its ability to align resource allocation with strategic objectives and operational needs. By adopting the assignment method, companies can achieve the following benefits:

  5. What is an Assignment Method?

    Assignment Method. In accounting and finance, the assignment method is a technique used for allocating or assigning resources, costs, or tasks among different departments, employees, or projects. The assignment method aims to optimize resource allocation to achieve maximum efficiency, cost savings, or other desired outcomes.

  6. Advantages and Disadvantages of Assignment Method Of Teaching

    Advantages of Assignment Method Of Teaching. Promotes independent learning - Assignment method of teaching encourages students to study and learn on their own, fostering self-reliance and self-learning.; Enhances critical thinking - This method also helps in developing critical thinking skills as students analyze and interpret the information themselves.

  7. Assignment problem

    The assignment problem is a fundamental combinatorial optimization problem. In its most general form, the problem is as follows: The problem instance has a number of agents and a number of tasks. Any agent can be assigned to perform any task, incurring some cost that may vary depending on the agent-task assignment.

  8. Understanding Assignments

    What this handout is about. The first step in any successful college writing venture is reading the assignment. While this sounds like a simple task, it can be a tough one. This handout will help you unravel your assignment and begin to craft an effective response. Much of the following advice will involve translating typical assignment terms ...

  9. Cost Allocation

    Cost allocation is the process of identifying, accumulating, and assigning costs to costs objects such as departments, products, programs, or a branch of a company. It involves identifying the cost objects in a company, identifying the costs incurred by the cost objects, and then assigning the costs to the cost objects based on specific criteria.

  10. Assignment Method

    The assignment method is a way of allocating organizational resources to projects and tasks. The assignment method can be used for many other purposes besides production allocations. It can be employed to assign the number of salespersons to a given territory or territories. It can also be used to match bidders to contracts and assign other ...

  11. Teaching Methods & Strategies

    Assignment. The assignment method of teaching is the most popular form of student-centered instruction. Assignments may include essays, research papers, oral presentations, projects, labs, or ...

  12. Assignment Problem: Meaning, Methods and Variations

    After reading this article you will learn about:- 1. Meaning of Assignment Problem 2. Definition of Assignment Problem 3. Mathematical Formulation 4. Hungarian Method 5. Variations. Meaning of Assignment Problem: An assignment problem is a particular case of transportation problem where the objective is to assign a number of resources to an equal number of activities so as to minimise total ...

  13. Advantages and Disadvantages of Assignment Method

    The 'Assignment Method' is a way to match things or people to tasks. It's like when a teacher gives each student a different question to answer. This method helps to get the best results by picking the right match. What are the advantages and disadvantages of Assignment Method. The following are the advantages and disadvantages of ...

  14. Types of Assignments and Assessments

    Types of Assignments and Assessments. Assignments and assessments are much the same thing: an instructor is unlikely to give students an assignment that does not receive some sort of assessment, whether formal or informal, formative or summative; and an assessment must be assigned, whether it is an essay, case study, or final exam.

  15. Assignment

    Assignment is a task given to students by a teacher or professor, usually as a means of assessing their understanding and application of course material. Assignments can take various forms, including essays, research papers, presentations, problem sets, lab reports, and more. Assignments are typically designed to be completed outside of class ...

  16. How to Solve the Assignment Problem: A Complete Guide

    The Hungarian method is an algorithm that uses a step-by-step approach to find the optimal assignment. The algorithm consists of the following steps: Subtract the smallest entry in each row from all the entries of the row. Subtract the smallest entry in each column from all the entries of the column. Draw the minimum number of lines to cover ...

  17. Chapter 15 Flashcards

    Study with Quizlet and memorize flashcards containing terms like What is a technique used to monitor jobs in progress? A. ConWIP cards B. Gantt load chart C. Gantt schedule chart D. assignment method, What focuses on developing varying schedules with the minimum number of workers? A. the assignment method B. finite capacity scheduling C. finite loading D. cyclical scheduling, Loading A. is the ...

  18. Cost assignment definition

    Assignment of direct costs. Direct costs can be traced directly to a cost object. For example, the valve used on a basketball is a direct cost of the basketball, since it is an item that is directly incorporated into the product. These costs are assigned to cost objects based on a bill of materials (in the case of materials) or a time sheet (in ...

  19. How to Solve an Assignment Problem Using the Hungarian Method

    In this lesson we learn what is an assignment problem and how we can solve it using the Hungarian method.

  20. Assignment Problem in Linear Programming : Introduction and Assignment

    Assignment problem is a special type of linear programming problem which deals with the allocation of the various resources to the various activities on one to one basis. It does it in such a way that the cost or time involved in the process is minimum and profit or sale is maximum. Though there problems can be solved by simplex method or by ...

  21. What is Assignment Problem

    Assignment Problem is a special type of linear programming problem where the objective is to minimise the cost or time of completing a number of jobs by a number of persons. The assignment problem in the general form can be stated as follows: "Given n facilities, n jobs and the effectiveness of each facility for each job, the problem is to ...

  22. Transportation and Assignment Models in Operations Research

    Transportation and assignment models are special purpose algorithms of the linear programming. The simplex method of Linear Programming Problems(LPP) proves to be inefficient is certain situations like determining optimum assignment of jobs to persons, supply of materials from several supply points to several destinations and the like.

  23. Difference Between Transportation Problem and Assignment Problem

    On the other hand, the assignment problem focuses on assigning tasks, jobs, or resources one-to-one. Both of these problems are usually solved through linear programming techniques. The transportation problem is commonly approached through simplex methods, and the assignment problem is addressed using specific algorithms like the Hungarian method.

  24. Entering Payment Methods for an Employee Assignment

    An assignment can have two instances of the same payment method, for example if salary is divided between two bank accounts. Employees with no personal payment method on record receive pay by the default payment method of their payrolls. You enter payment methods for employee assignments in the Personal Payment Method window.

  25. Does the Scandinavian sleep method work? We put it to the test.

    The idea: Two people share a bed but use separate covers. The promise: a better night's sleep. The bedding arrangement is popular in other European countries as well; on a trip to Paris a couple ...

  26. Transformer-based assignment decision network for multiple object

    To generate complete trajectories such methods employ a data association process to establish assignments between detections and existing targets during each timestep. Recent data association approaches try to solve either a multi-dimensional linear assignment task or a network flow minimization problem or tackle it via multiple hypotheses ...